Suppose a car gas tank holds 21 gallons of gas. On a trip it uses 2/3 of a tank of gas on a trip. Then it uses 1/3 of what is left in the tank. How many gallons of gas are left in the tank? If u can could u show the work?

Answers

Answer 1

9514 1404 393

Answer:

  2/9 tank

Step-by-step explanation:

After the first trip, 1 -2/3 = 1/3 of the tank remains.

After the second trip, 1 -1/3 = 2/3 of that remains.

After both trips, (1/3 tank)(2/3) = 2/9 tank remains.


Related Questions

que es un plano cartesiano?

Answers

Answer: Un sistema de coordenadas cartesianas en un plano es un sistema de coordenadas que especifica cada punto de forma única mediante un par de coordenadas numéricas, que son las distancias con signo al punto desde dos líneas fijas orientadas perpendiculares, medidas en la misma unidad de longitud. Step-by-step explanation:

What is the sum of the measures of the interior angles of any triangle?
100°
180°
270°
360°

Answers

I think the answer is 180

Answer:

180°

Step-by-step explanation:

E D G E N U I T Y

Hope this helped! :)

Stay safe and have a wonderful day/night!!!!!

Please give Brainliest to the person who answered before me.


[tex](5 - x)(x - 3)[/tex]

Answers

Answer:

15x

Step-by-step explanation:

because i multiply i side by side and copy the variable

Answer:

- x² + 8x - 15

Step-by-step explanation:

Given

(5 - x)(x - 3)

Each term in the second factor is multiplied by each term in the first factor, that is

5(x - 3) - x(x - 3) ← distribute parenthesis

= 5x - 15 - x² + 3x ← collect like terms

= - x² + 8x - 15 ← in standard form

plzz help with this math due today

Answers

Answer:

20x⁴ - 13x³+ 8x² - x + 6

Step-by-step explanation:

Multiplyin gbthebtop trinomial by the last term. Of the bottom trinomial.

___4x² - 5x + 3

___5x² + 3x + 2

* ____________

_________8x² - 10x + 6

______12x³-15x²+9x

_20x⁴-25x³+15x²

____________________

20x⁴ - 13x³+ 8x² - x + 6

___________________

4 points
The midpoint of AB is (-1,5)
and the coordinates of point A are
(-3,2). What are the coordinates
of point B?

Answers

Answer:

someone go answer my question plssssssssssssssssssssssssss

Step-by-step explanation:

Find the value of x using the exterior angle property in the figure below (label the diagram

Answers

Answer:

x = 72°

Step-by-step explanation:

x + 38° + 70° = 180°

x + 108° = 108°

x = 72°

Answer:

x is 32

This is your answer.

Hope it helps!!!

hi hi hi hi it’s b right?

Answers

Answer:B

Step-by-step explanation: yes b as the x is always 50x greater than the Y

Answer: yes it’s B. My dear!

Step-by-step explanation: hope this helps :)

I have to find the area of this rectangular prism pls help!!

Answers

Answer:

the surface area is 208

Step-by-step explanation:

formula for surface area is A = 2 (w*l + h*l + h*w )

A = 2 ( 4*8 + 6*8 + 6*4 )

A = 2 ( 32 + 48 + 24 )

A = 2 ( 104 )

A = 208

Solve the equation and check the solution.
1 1
O a=1
1
O a = 3
2
O a=4
o
1
a = = 4
2

Answers

[tex]\huge\text{Hey there!}[/tex]

[tex]\large\text{Equation: }\mathsf{a - 2\dfrac{1}{2} = 1\dfrac{1}{2}}[/tex]

[tex]\large\text{You have to SIMPLIFY the BOTH SIDES of the given EQUATION}[/tex]

[tex]\mathsf{-2\dfrac{1}{2}=\bf -\dfrac{5}{2}}[/tex]

[tex]\mathsf{1\dfrac{1}{2}=\bf \dfrac{3}{2}}[/tex]

[tex]\large\text{New equation: }\mathsf{a + \dfrac{-5}{2}= \dfrac{3}{2}}[/tex]

[tex]\large\text{ADD by }\mathsf{\dfrac{5}{2}}\large\text{ to BOTH SIDES}[/tex]

[tex]\mathsf{a + \dfrac{-5}{2}+\dfrac{5}{2}= \dfrac{3}{2}+ \dfrac{5}{2}}[/tex]

[tex]\large\text{Cancel out: }\mathsf{-\dfrac{5}{2}+ \dfrac{5}{2}}\large\text{ because it gives you 1}[/tex]

[tex]\large\text{Keep: }\mathsf{\dfrac{3}{2}+\dfrac{5}{2}}\large\text{ because it helps solve for \bf a}[/tex]

[tex]\large\text{New equation: }\mathsf{a = \dfrac{3}{2}+\dfrac{5}{2}}[/tex]

[tex]\mathsf{\dfrac{3+5}{2\ \& \ 2}}[/tex]

[tex]\mathsf{3 + 5 = 8}[/tex]

[tex]\mathsf{2\ \& \ 2=2}[/tex]

[tex]\large\text{Equation: }\mathsf{a = \dfrac{8}{2}}[/tex]

[tex]\large\text{Simplify above and you have the \bf a value}\uparrow[/tex]

[tex]\boxed{\boxed{\large\textsf{Answer: \huge \bf a = 4}}}\huge\checkmark[/tex]

[tex]\text{Good luck on your assignment and enjoy your day!}[/tex]

~[tex]\frak{Amphitrite1040:)}[/tex]

HELP PLEASEE PLEASE PLEASE!! It’s due by 10:20 !!

A cone has a height that is greater than the diameter of the base of the cone.
What is the shape of a cross section of the cone that contains the vertex of the cone and the diameter of the base?
А
right triangle
B.
obtuse triangle
С
scalene triangle
D
isosceles triangle

Answers

Answer:

its either b or d

In a shipment of 400 parts, 14 are found to be defective. How many defective parts should be expected in a shipment of 1200?

Answers

1200 parts/ 400 parts = 3

12 parts x 3 = 36 parts

In 1200 parts, 36 would be bad.

Find the slope of the line that passes through A(0,−5) and B(3,−4) .

Answers

Answer: 1/3

Step-by-step explanation: slope is the difference between the y values over the difference of the x values

The slope of the line that passes through A(0,−5) and B(3,−4) will be 1/3.

What is slope?

A slope is a tangent or angle at a point and a slope is the intensity of inclination of any geometrical lines.

Slope = Tanx where x will be the angle from the positive x-axis at that point.

The slope could be positive or negative.

The slope associated with two points (x₁, y₁) and (x₂, y₂) is given by

Slope = (y₂ - y₁)/(x₂ - x₁)

Slope m = (-4 + 5)/(3 - 0)

m = 1/3

Hence "The slope of the line that passes through A(0,−5) and B(3,−4) will be 1/3".

To learn more about slopes,

https://brainly.com/question/3605446

#SPJ2

Paul opens a savings account with $350. He saves $150 per month. After how many months will Paul have $2150 in the account?​

Answers

Answer:

12 months

Step-by-step explanation:

2150-350=1800

1800÷150=12

1.) What is the equation given a slope of ½ and a y-intercept of 4?

2.) What is the slope of an equation y=2x-5

3.) What is the y-intercept of an equation y=7x+8?​

Answers

1) y=1/2x+4
2) slope is 2
3) 8

A unicycle wheel has a diameter of 12 inches. How many inches will the unicycle
travel in 5 revolutions?
Use n = 3.14 and round your answer to the nearest hundredth of an inch.

Answers

Answer:The unicycle would travel 226.29 inches in 6 revolutions.

Step-by-step explanation:

I have a math test in an hour and a half. It’s about algebraic word problems. Can someone help me. I’m really struggling in maths and might fail

Answers

Answer:

yes I can try to help you what I can

Algebraic Expressions Word Problems -
WORD PROBLEMS require practice in translating verbal language into algebraic language. See Lesson 1, Problem 8. Yet, word problems fall into distinct types. Below are some examples.
Example 1. ax ± b = c. All problems like the following lead eventually to an equation in that simple form.
Jane spent $42 for shoes. This was $14 less than twice what she spent for a blouse. How much was the blouse?
Solution. Every word problem has an unknown number. In this problem, it is the price of the blouse. Always let x represent the unknown number. That is, let x answer the question.
Let x, then, be how much she spent for the blouse. The problem states that "This" -- that is, $42 -- was $14 less than two times x.
Here is the equation:
2x − 14 = 42.

2x = 42 + 14 (Lesson 9)

= 56.

x = 56
2

= 28.
The blouse cost $28.
Example 2. There are b boys in the class. This is three more than four times the number of girls. How many girls are in the class?
Solution. Again, let x represent the unknown number that you are asked to find: Let x be the number of girls.
(Although b is not known -- it is an arbitrary constant -- it is not what you are asked to find.)
The problem states that "This" -- b -- is three more than four times x:
4x + 3 = b.
Therefore,
4x = b − 3

x = b − 3
4 .
The solution here is not a number, because it will depend on the value of b. This is a type of "literal" equation, which is very common in algebra.
Example 3. The whole is equal to the sum of the parts.
The sum of two numbers is 84, and one of them is 12 more than the other. What are the two numbers?
Solution. In this problem, we are asked to find two numbers. Therefore, we must let x be one of them. Let x, then, be the first number.
We are told that the other number is 12 more, x + 12.
The problem states that their sum is 84:
word problem = 84
The line over x + 12 is a grouping symbol called a vinculum. It saves us writing parentheses.
We have:
2x = 84 − 12

= 72.

x = 72
2

= 36.
This is the first number. Therefore the other number is
x + 12 = 36 + 12 = 48.
The sum of 36 + 48 is 84.
Example 4. The sum of two consecutive numbers is 37. What are they?
Solution. Two consecutive numbers are like 8 and 9, or 51 and 52.
Let x, then, be the first number. Then the number after it is x + 1.
The problem states that their sum is 37:
word problem = 37
2x = 37 − 1

= 36.

x = 36
2

= 18.
The two numbers are 18 and 19.
Example 5. One number is 10 more than another. The sum of twice the smaller plus three times the larger, is 55. What are the two numbers?
Solution. Let x be the smaller number.
Then the larger number is 10 more: x + 10.
The problem states:
2x + 3(x + 10) = 55.
That implies
2x + 3x + 30 = 55. Lesson 14.

5x = 55 − 30 = 25.

x = 5.
That's the smaller number. The larger number is 10 more: 15.

100 POINTS AND BRAINLIEST! Explain your answer PLEASE!

Answers

Answer:

27000

Step-by-step explanation:

I = P×R×T

=2000×25/10×5

=25000

AMOUNT= P+I

=2000+25000

=27000

Compound interest:-

P(1+r)^t2000(1+0.025)⁵2000(1.025)⁵2000(1.314)2628$

Total:-

2000+2628=4628$

An African Elephant can eat 4,620 pounds of
food a week! If the elephant ate the same
amount of food each day that week, how many
pounds of food would he have eaten in a day?

Answers

Answer:

Step-by-step explanation:

660

Answer:

660 pounds

Step-by-step explanation:

(b) A farmer uses of his land to plant
cassava, of the remaining land to plant
maize and the rest vegetables. If
vegetables cover an area of 10 arces, what
is the total area of the farmer's land.​

Answers

Answer:

Cassava = ⅓ or 33.333%

Maize = ⅔x⅔ = 44.444%

total so far =77.777%

Remaining vegetable area =(100–77.777%) = 22.222%

If remaining area vegetables = 22.222% = 10 acres

Maize = ⅔x⅔ = 44.444% or 20 acres

Cassava = ⅓ or 33.333% or 15 acres

Total land area = 45 acres.

Step-by-step explanation:

Suppose a value of 13 is added to the existing values 17,13,15,26,23,29,18,22,15. Find the median of the values. ​

Answers

Answer:

17.5

Step-by-step explanation:

Given values:

17, 13, 15, 26, 23, 29, 18, 22, 15 and added 13

Put the values in ascending order:

13, 13, 15, 15, 17, 18, 22, 23, 26, 29

There are even values, the middle numbers are:

17 and 18

Median is the average of the middle two numbers:

(17 + 18)/2 = 17.5

Predict what will happen to the lynx population after 1935. Will it go up or down? Explain why you think this.
Please help me. I will adore u

Answers

Answer:

It'll go down

because they're very rare

At a hospital, 56 percent of the babies born are not girls. Of the baby girls born, 12 percent are premature. What is the probability of a premature baby girl being born at this hospital? Round to the nearest percent.

5%
7%
21%
27%

Answers

Answer:

It's the first option 5%

Step-by-step explanation:

The probability of a premature baby girl being born at this hospital is,

⇒ 5%

What is mean by Probability?

The term probability refers to the likelihood of an event occurring. Probability means possibility. It is a branch of mathematics that deals with the occurrence of a random event. The value is expressed from zero to one

Now, If the likelihood of boys being born is 56%,

Then, the complement of that is the likelihood of girls being born which is,

⇒ 100% - 56% = 44%.

Since, we are given that of the baby girls born, 12% are premature, the probability of a premature baby girl  being born is:

⇒ (0.44)(0.12)

= 0.0528

⇒ 5.28%

Thus, The probability of a premature baby girl being born at this hospital is,

⇒ 5%

Learn more about the probability visit:

https://brainly.com/question/13604758

#SPJ7

which of the following functions is graphed below?​

Answers

You need to show the rest of the functions

please help im struggling and it’s due in 30 minutes

Answers

i know this is not an answer but if your having trouble with this there is a website called Desmos all you do is type in the equation for the graph and it gives you the answer

Solve each equation. 15 = t - 25 m6 = 18 5d = 75 94 = g + 32

Answers

Answer:

1, t = 40

2, m = 3

3, d = 15

4, g = 62

Step-by-step explanation:

15 = t - 25

+25.     +25

40 = t

M6/6 = 18/6

m = 3

5d/5 = 75/5

d = 15

94 = g + 32

-32.      -32

62 = g

What is the answer to
-k - (-8k)

Answers

Answer:

if we simplify then the answer is 7k

Step-by-step explanation:

can someone please answer my question from my profile.

Answer:

7k

Step-by-step explanation:

The table below shows all of the possible outcomes for rolling two six-sided number cubes. 1 2 First Number Cube Second Number Cube 1 2 3 4 5 6 1,1 1,2 1,3 1,4 1,5 1,6 2,1 2,2 2,3 2,4 2,5 2,6 3,1 3,2 3,3 3,4 3,5 3,6 4,1 4,2 4,3 4,4 4,5 4,6 5,1 5,2 5,3 5,4 5,5 5,6 6,1 6,2 6,3 6,4 6,5 6,6 4 5 What is the probability of rolling an even number first and an odd number second?
Answer choices:
A. 1/9,
B. 1/6
C. 1/4
D.1/2​

Answers

Answer:

(1/6)

Step-by-step explanation:

ℎ 1/6

does anyone know how to solve this? pls leave instructions

Answers

Answer:

7.5

Step-by-step explanation:

Divide 144g in the ratio 7: 11​

Answers

Answer:

88:56 = 11:7

Step-by-step explanation:

Step One

Add the two ratios together 11 + 7 = 18

Now set up a ratio to solve for 11 parts out of 18

11:18 = x:144         Proportion

11 / 18 = x / 144    Equation: Cross mulitiply

11*144 = 18x         Combine the left side

1584   = 18x         Divide by 18

1584 / 18  = x

88 = x

That is the larger amount (11 is larger than 7)

Find the smaller amount

144 - 88 = 56

The two amounts are in the ratio of 11:7

88:56 = 11:7  

I hopes this helps a little  bit

Answer:

56g and 88g

Step-by-step explanation:

7x+11x=144

18x=144

x=144/18

x=8

7x=7*8=56g

11x=11*8=88g

what about these 2 ?

Answers

Answer:

11. A) 12

12. D)11

Step-by-step explanation:

11. We substitute x with 12 in 3(x)-11.

3(12)-11=

36-11= 25

We substitute x with 12 for the second equation (x+13).

12+13=25

Both answers match so the answer for #11 is 12.

12.We substitute x with 11 in 4(x)-4.

4(11)-4=

44-4=40

We substitute x with 11 for the second equation (2(x)+18)

2(11)+18=

22+18=40

Both answers match so the answer for #12 is 11.

Other Questions
please heLP I dont know any of this sadly. Saturn Corporation issued $300,000 par value 10-year bonds at 107 on January 1, 20X3, which Star Corporation purchased. Pluto Corporation owns 65% of Saturn's voting shares. On Jan 1, 20X7, Pluto Corporation purchased $120,000 face value of Saturn bonds from Star for $118,020. On the date Pluto purchased the bonds, the bonds' carrying value on Saturn's book was $126,019. The bonds pay 12 percent interest annually on December 31. The preparation of consolidated financial statements for Saturn and Pluto at December 31, 20X9, required the following consolidating entry: Based on the information given above, what amount of gain or loss on bond retirement is included in the 20X7 consolidated income statement? a. $8,000 gain b. $5,200 gain c. $8,000 loss d. $5,200 loss Which improper fraction is equivalent to 15 3/7 how did the hindus and guptas relate to ancient india economically What is the median?A(18B(27C(33.2D(34.5 what kingdom does eubacteria belong to 5. Choose the likelihood of a probability of 50 percent.Event: November 25th will fall on a MondayCertainHighly likelyLess likelyImpossible (20 POINTS) Will Give Brainliest If You Explain Your Answer:The table represents the linear function f(x), and the equation represents the linear function g(x).Compare the y-intercepts and slopes of the linear functions f(x) and g(x) and choose the answer that best describes them.x f(x)0 12 94 17g(x) = 3x + 1A. The slope of f(x) is less than the slope of g(x). The y-intercept of f(x) is equal to the y-intercept of g(x).B. The slope of f(x) is less than the slope of g(x). The y-intercept of f(x) is greater than the y-intercept of g(x).C. The slope of f(x) is greater than the slope of g(x). The y-intercept of f(x) is equal to the y-intercept of g(x).D. The slope of f(x) is greater than the slope of g(x). The y-intercept of f(x) is greater than the y-intercept of g(x). David has 54 marbles, 24 of which are black, 18 are red, and 12 are blue. What fraction represents the ratio of black marbles to blue marbles? How did Grigori Rasputin work for the Romanov family? Why is fluorine more chemically reactive than nitrogen given the equation y=5x-1 does the point A(-11,54) belong to this line? Can someone please write a short paragraph about karma? (8 points)I need this done today! I will mark brainliest The snack that smiles back ____ PLEASE HELP! Tammy and her team of surgeons are creating procedures So far, they'veidentified areas that need improvement and listed goals What is the nextstep the team should take?A. Drafta procedures plan and designate task for each teammemberB. Discuss team goals and ask forcach team member to hare theiropinionsC. Strategize solutions for implementing the procedureD. Conduct research to collect data that relie to withreach their goals Now that you have worked through a lot of material that includes these basic patterns, and you have compared grammatically correct and incorrect sentences, write down what you think is a rule that could explain what makes a sentence grammatically correct or not. For example, you might write something like: "verbs always match nouns in number, and they usually come before the noun." In other words, make your best guess for the grammar rule that makes sense out of the pattern(s) you see in the phrases you have been working with. Review if you need to, and you might briefly check your hunches against the sentences you have been working with in this or previous modules. Keep in mind that what you're after is your hunch, not a grammar rule from a text book. Now check your hunch with the explanation of this principle in the following pattern. I need help thank you Where are chromosomes found in the human body what are chromosomes made of Bianca makes an error when she tries to write an expression equivalent to12 + 15(3 - y) - 10y. What is the error? Fix Bianca's error.I NEED HELP ASAPPPPPP!!! What color is a carrot